Đến nội dung

Hình ảnh

$\boxed{\text{Chuyên Đề}}$ Bất đẳng thức - Cực trị


  • Please log in to reply
Chủ đề này có 742 trả lời

#181
Hoang Tung 126

Hoang Tung 126

    Thiếu tá

  • Thành viên
  • 2061 Bài viết

58) Cho $x\in (0;1)$. Tìm Min $A=\frac{4x^2+1}{x^2(1-x)}$

 

59) Cho $x\in (0;3)$. Tìm Max $B=(5x^2-14x-3)(x-3)$

 

60) Cho $a;b;c$ là độ dài 3 cạnh một tam giác. Cmr: $\sum \frac{a}{\sqrt{b+c-a}}\geq \sum \sqrt{a}$

 

61) Cho $a;b;c$ là độ dài 3 cạnh một tam giác. Cmr: $|\sum \frac{a-b}{a+b}|<\frac{1}{8}$

 

62) Cho $x\in [0;1]$. Cmr: $\sqrt[4]{1-x^2}+\sqrt[4]{1-x}+\sqrt[4]{1+x}\leq 3$

 

63) Cho $a;b;c>0$. Cmr: $\frac{1}{a^3}+\frac{a^3}{b^3}+b^3\geq \frac{1}{a}+\frac{a}{b}+b$

 

64)

a) Cho $x\in [-1;1]$. Cmr: $|4x^3-3x|\leq 1$

b) Cho $\left\{\begin{matrix}a_1;a_2;...;a_n\in [-1;1] & & \\ a_1^3+a_2^3+...+a_n^3=0 & & \end{matrix}\right.$. Cmr: $a_1+a_2+...+a_n\leq \frac{n}{3}$

 

65) Cho $x;y>0$ thỏa $x^2+y^2=1$. Cmr: $xy+Max(x;y)\leq \frac{3\sqrt{3}}{4}$ (Giải thích: Nếu $x>y$ thì $Max(x;y)=x$ và tương tự)

 

 

P/s: Anh Daicagiangho1998 học KHTN nên cứ từ từ mà làm thôi, chứ có đề phát đã ăn hết sạch luôn vậy :D

Bài 62:Theo AM-GM 4 số có:$\sqrt[4]{1-x^2}=\sqrt[4]{(1-x)(1+x).1.1}\leq \frac{1-x+1+x+1+1}{4}=1$

                                             $\sqrt[4]{(1-x).1.1.1}\leq \frac{1-x+1+1+1}{4}=\frac{4-x}{4}$

                                             $\sqrt[4]{(1+x).1.1.1}\leq \frac{1+x+1+1+1}{4}=\frac{4+x}{4}$

Cộng theo vế các bdt $= > \sqrt[4]{1-x^2}+\sqrt[4]{1+x}+\sqrt[4]{1-x}\leq \frac{4-x+4+x}{4}+1=3$



#182
Hoang Tung 126

Hoang Tung 126

    Thiếu tá

  • Thành viên
  • 2061 Bài viết

58) Cho $x\in (0;1)$. Tìm Min $A=\frac{4x^2+1}{x^2(1-x)}$

 

59) Cho $x\in (0;3)$. Tìm Max $B=(5x^2-14x-3)(x-3)$

 

60) Cho $a;b;c$ là độ dài 3 cạnh một tam giác. Cmr: $\sum \frac{a}{\sqrt{b+c-a}}\geq \sum \sqrt{a}$

 

61) Cho $a;b;c$ là độ dài 3 cạnh một tam giác. Cmr: $|\sum \frac{a-b}{a+b}|<\frac{1}{8}$

 

62) Cho $x\in [0;1]$. Cmr: $\sqrt[4]{1-x^2}+\sqrt[4]{1-x}+\sqrt[4]{1+x}\leq 3$

 

63) Cho $a;b;c>0$. Cmr: $\frac{1}{a^3}+\frac{a^3}{b^3}+b^3\geq \frac{1}{a}+\frac{a}{b}+b$

 

64)

a) Cho $x\in [-1;1]$. Cmr: $|4x^3-3x|\leq 1$

b) Cho $\left\{\begin{matrix}a_1;a_2;...;a_n\in [-1;1] & & \\ a_1^3+a_2^3+...+a_n^3=0 & & \end{matrix}\right.$. Cmr: $a_1+a_2+...+a_n\leq \frac{n}{3}$

 

65) Cho $x;y>0$ thỏa $x^2+y^2=1$. Cmr: $xy+Max(x;y)\leq \frac{3\sqrt{3}}{4}$ (Giải thích: Nếu $x>y$ thì $Max(x;y)=x$ và tương tự)

 

 

P/s: Anh Daicagiangho1998 học KHTN nên cứ từ từ mà làm thôi, chứ có đề phát đã ăn hết sạch luôn vậy :D

Bài 64:Ta có:$\left | 4x^3-3x \right |\leq 1< = > -1\leq 4x^3-3x\leq 1< = > 4x^3-3x+1\geq 0,1+3x-4x^3\geq 0< = > (2x-1)^2(x+1)\geq 0,(1-x)(4x^2-4x+1)\geq 0$(Luôn đúng theo giả thiết)

 

 

 


#183
Hoang Tung 126

Hoang Tung 126

    Thiếu tá

  • Thành viên
  • 2061 Bài viết

58) Cho $x\in (0;1)$. Tìm Min $A=\frac{4x^2+1}{x^2(1-x)}$

 

59) Cho $x\in (0;3)$. Tìm Max $B=(5x^2-14x-3)(x-3)$

 

60) Cho $a;b;c$ là độ dài 3 cạnh một tam giác. Cmr: $\sum \frac{a}{\sqrt{b+c-a}}\geq \sum \sqrt{a}$

 

61) Cho $a;b;c$ là độ dài 3 cạnh một tam giác. Cmr: $|\sum \frac{a-b}{a+b}|<\frac{1}{8}$

 

62) Cho $x\in [0;1]$. Cmr: $\sqrt[4]{1-x^2}+\sqrt[4]{1-x}+\sqrt[4]{1+x}\leq 3$

 

63) Cho $a;b;c>0$. Cmr: $\frac{1}{a^3}+\frac{a^3}{b^3}+b^3\geq \frac{1}{a}+\frac{a}{b}+b$

 

64)

a) Cho $x\in [-1;1]$. Cmr: $|4x^3-3x|\leq 1$

b) Cho $\left\{\begin{matrix}a_1;a_2;...;a_n\in [-1;1] & & \\ a_1^3+a_2^3+...+a_n^3=0 & & \end{matrix}\right.$. Cmr: $a_1+a_2+...+a_n\leq \frac{n}{3}$

 

65) Cho $x;y>0$ thỏa $x^2+y^2=1$. Cmr: $xy+Max(x;y)\leq \frac{3\sqrt{3}}{4}$ (Giải thích: Nếu $x>y$ thì $Max(x;y)=x$ và tương tự)

 

 

P/s: Anh Daicagiangho1998 học KHTN nên cứ từ từ mà làm thôi, chứ có đề phát đã ăn hết sạch luôn vậy :D

Bài 65:Bài này khá hay:Giả sử $x=Max(x,y)$

 Theo AM-GM có:$xy\leq \frac{x^2+3y^2}{2\sqrt{3}},x\leq \frac{4x^2+3}{4\sqrt{3}}$

$= > xy+max(x,y)=x+y\leq \frac{2(x^2+3y^2)+(4x^2+3)}{4\sqrt{3}}=\frac{6(x^2+y^2)+3}{4\sqrt{3}}=\frac{3\sqrt{3}}{4}$

Dấu = xảy ra tại $x=\frac{\sqrt{3}}{2},y=\frac{1}{2}$



#184
Viet Hoang 99

Viet Hoang 99

    $\textbf{Trương Việt Hoàng}$

  • Điều hành viên THPT
  • 2291 Bài viết

58) Cho $x\in (0;1)$. Tìm Min $A=\frac{4x^2+1}{x^2(1-x)}$

 

59) Cho $x\in (0;3)$. Tìm Max $B=(5x^2-14x-3)(x-3)$

 

60) Cho $a;b;c$ là độ dài 3 cạnh một tam giác. Cmr: $\sum \frac{a}{\sqrt{b+c-a}}\geq \sum \sqrt{a}$

 

61) Cho $a;b;c$ là độ dài 3 cạnh một tam giác. Cmr: $|\sum \frac{a-b}{a+b}|<\frac{1}{8}$

 

62) Cho $x\in [0;1]$. Cmr: $\sqrt[4]{1-x^2}+\sqrt[4]{1-x}+\sqrt[4]{1+x}\leq 3$

 

63) Cho $a;b;c>0$. Cmr: $\frac{1}{a^3}+\frac{a^3}{b^3}+b^3\geq \frac{1}{a}+\frac{a}{b}+b$

 

64)

a) Cho $x\in [-1;1]$. Cmr: $|4x^3-3x|\leq 1$

b) Cho $\left\{\begin{matrix}a_1;a_2;...;a_n\in [-1;1] & & \\ a_1^3+a_2^3+...+a_n^3=0 & & \end{matrix}\right.$. Cmr: $a_1+a_2+...+a_n\leq \frac{n}{3}$

 

65) Cho $x;y>0$ thỏa $x^2+y^2=1$. Cmr: $xy+Max(x;y)\leq \frac{3\sqrt{3}}{4}$ (Giải thích: Nếu $x>y$ thì $Max(x;y)=x$ và tương tự)

 

 

P/s: Anh Daicagiangho1998 học KHTN nên cứ từ từ mà làm thôi, chứ có đề phát đã ăn hết sạch luôn vậy :D

 

 

Đặt $x^2=b+c-a;y^2=a+c-b;z^2=a+b-c$

$\Rightarrow a=\frac{y^2+z^2}{2};b=\frac{x^2+z^2}{2};c=\frac{x^2+y^2}{2}$

BĐT cần cm trở thành $\sum \frac{x^2+y^2}{2x}\geq \sum \sqrt{\frac{x^2+y^2}{2}}$

Áp dụng bđt Cauchy-Shwarz có

$\sum \frac{x^2+y^2}{2x}$ $\geq \frac{(\sum\sqrt{\frac{x^2+y^2}{2}})^2 }{x+y+z}$

Bằng $AM-GM$ ta dễ chứng minh $\sum x\leq \sum \sqrt{\frac{x^2+y^2}{2}}$

$\Rightarrow \sum \frac{x^2+y^2}{2x}\geq \sum\sqrt{\frac{x^2+y^2}{2}}$ (đpcm)

Chỗ kia là $\sum \frac{y^2+z^2}{2x}$ nhé.

60)
Cách 2:

Áp dụng Cauchy:
$\sum \frac{a}{\sqrt{b+c-a}}+\sum \sqrt{b+c-a}\geq \sum 2\sqrt{a}$

Áp dụng BCS:
$\sqrt{b+c-a}+\sqrt{c+a-b}\leq \sqrt{2(b+c-a+c+a-b)}=2\sqrt{c}\Rightarrow \sum \sqrt{b+c-a}\leq \sum \sqrt{a}$

Trừ 2 vế của 2 BĐT ngược chiều có đpcm.

Dấu = xảy ra khi: $a=b=c$

 

 

Bài 62:Theo AM-GM 4 số có:$\sqrt[4]{1-x^2}=\sqrt[4]{(1-x)(1+x).1.1}\leq \frac{1-x+1+x+1+1}{4}=1$

                                             $\sqrt[4]{(1-x).1.1.1}\leq \frac{1-x+1+1+1}{4}=\frac{4-x}{4}$

                                             $\sqrt[4]{(1+x).1.1.1}\leq \frac{1+x+1+1+1}{4}=\frac{4+x}{4}$

Cộng theo vế các bdt $= > \sqrt[4]{1-x^2}+\sqrt[4]{1+x}+\sqrt[4]{1-x}\leq \frac{4-x+4+x}{4}+1=3$

62) Dấu = xảy ra khi: $x=0$

 

 

Bài 64:

a) Ta có:$\left | 4x^3-3x \right |\leq 1< = > -1\leq 4x^3-3x\leq 1< = > 4x^3-3x+1\geq 0,1+3x-4x^3\geq 0< = > (2x-1)^2(x+1)\geq 0,(1-x)(4x^2-4x+1)\geq 0$(Luôn đúng theo giả thiết)

Dấu = xảy ra khi: $x=1$ hoặc $x=-1$

64)

 

b) Áp dụng câu a.

$\left\{\begin{matrix}|4a_1^3-3a_1|\leq 1 & & \\ ... & & \\ |4a_n^3-3a_n|\leq 1 \end{matrix}\right.\Rightarrow |4a_1^3-3a_1|+...+|4a_n^3-3a_n|\leq |4(a_1^3+...+a_n^3)-3(a_1+...+a_n)|\leq n\Leftrightarrow a_1+...+a_n\leq \frac{n}{3}$

 


Bài 65:Bài này khá hay:Giả sử $x=Max(x,y)$

 Theo AM-GM có:$xy\leq \frac{x^2+3y^2}{2\sqrt{3}},x\leq \frac{4x^2+3}{4\sqrt{3}}$

$= > xy+max(x,y)=$ $x+y$ $\leq \frac{2(x^2+3y^2)+(4x^2+3)}{4\sqrt{3}}=\frac{6(x^2+y^2)+3}{4\sqrt{3}}=\frac{3\sqrt{3}}{4}$

Dấu = xảy ra tại $x=\frac{\sqrt{3}}{2},y=\frac{1}{2}$

 

Chỗ kia là $x+xy$

65)
Cách 2:

Giả sử $x=Max(x;y)$

$\Rightarrow xy+Max(x;y)=xy+x\leq \frac{3\sqrt{3}}{4}\Rightarrow x(y+1)\leq \frac{3\sqrt{3}}{4}\Leftrightarrow x^2(y+1)^2\leq \frac{27}{16}\Rightarrow (1-y^2)(y+1)^2\leq \frac{27}{16}\Leftrightarrow (1-y)(y+1)^3\leq \frac{27}{16}\Leftrightarrow (3-3y)(y+1)(y+1)(y+1)\leq \frac{81}{16}$ (*)

Mà $(3-3y)(y+1)(y+1)(y+1)\leq (\frac{(3-3y+y+1+y+1+y+1)}{4})^4=\frac{81}{16}$

Vậy (*) luôn đúng.

Dấu = xảy ra tại $x=\frac{\sqrt{3}}{2},y=\frac{1}{2}$


Bài viết đã được chỉnh sửa nội dung bởi Viet Hoang 99: 14-02-2014 - 11:22


#185
nguyen huy nam

nguyen huy nam

    Lính mới

  • Thành viên
  • 4 Bài viết

Lời giải:

a)$(x-2y)^{2}+3(x+1)^{2}\geq 0$ (luôn đúng)

Dấu "=" xảy ra khi: $x=-1;y=\frac{-1}{2}$.

b)$a^{3}(a-b)-b^{3}(a-b)\geq 0$

$\Leftrightarrow (a-b)^{2}(a^{2}-ab+b^{2})\geq 0$ (luôn đúng)

c)$(x-1)^{2}+(2y-3)^{2}+3(z-1)^{2}+1>0$ (luôn đúng)
d)$\frac{1}{2}[(a-b)^{2}+(b-c)^{2}+(c-a)^{2}]\geq 0$ (luôn đúng)

e) Áp dụng câu d)
$a^{2}+b^{2}+c^{2}\geq ab+bc+ca$

$\Leftrightarrow 2(a^{2}+b^{2}+c^{2})\geq 2(ab+bc+ca)$

$\Leftrightarrow 3(a^{2}+b^{2}+c^{2})\geq (a+b+c)^{2}$ (cộng 2 vế với $a^{2}+b^{2}+c^{2}> 0$)

g) Đặt $\sqrt[3]{a}=x>0$

$PT\Leftrightarrow 1+x^{3}\geq x+x^{2}$

$\Leftrightarrow (x-1)^{2}(x+1)\geq 0$ (luôn đúng do $x>0$)

h) $\Leftrightarrow \frac{a\sqrt{a}+b\sqrt{b}}{\sqrt{ab}}\geq \sqrt{a}+\sqrt{b}$

$\Leftrightarrow a\sqrt{a}+b\sqrt{b}-a\sqrt{b}-b\sqrt{a}\geq 0$

$\Leftrightarrow (a-b)(\sqrt{a}-\sqrt{b})\geq 0$

$\Leftrightarrow (\sqrt{a}-\sqrt{b})^{2}(\sqrt{a}+\sqrt{b})\geq 0$ (luôn đúng do $a;b>0$)

mình ko hiểu những gì bạn làm lắm



#186
Viet Hoang 99

Viet Hoang 99

    $\textbf{Trương Việt Hoàng}$

  • Điều hành viên THPT
  • 2291 Bài viết

58) Cho $x\in (0;1)$. Tìm Min $A=\frac{4x^2+1}{x^2(1-x)}$

 

59) Cho $x\in (0;3)$. Tìm Max $B=(5x^2-14x-3)(x-3)$

 

61) Cho $a;b;c$ là độ dài 3 cạnh một tam giác. Cmr: $|\sum \frac{a-b}{a+b}|<\frac{1}{8}$

 

58)

$\frac{4x^2+1}{x^2(1-x)}=\frac{4x+\frac{1}{x}}{x(1-x)}\geq \frac{2\sqrt{4}}{(\frac{x+1-x}{2})^2}=\frac{4}{\frac{1}{4}}=16$

Dấu = xảy ra khi: $x=\frac{1}{2}$

59)

$B=(5x+1)(x-3)(x-3)=(5x+1)(3-x)(3-x)$

Từ $GT\Rightarrow \left\{\begin{matrix}5x+1>0 & & \\ 3-x>0 & & \end{matrix}\right.$

$B=(5x+1)(3-x)(3-x)=\frac{1}{50}(10x+2)(15-5x)(15-5x)\leq \frac{1}{50}(\frac{10x+2+15-5x+15-5x}{3})^3=\frac{1}{50}.(\frac{32}{3})^3=\frac{2^{14}}{5^2.3^3}$

Dấu = xảy ra khi: $x=\frac{13}{15}$

61)

Ta sẽ CM:
$\sum \frac{a-b}{a+b}=\frac{(a-b)(b-c)(a-c)}{(a+b)(b+c)(c+a)}$ (1)

$\Leftrightarrow (a-b)(b+c)(c+a)+(b-c)(a+b)(c+a)+(c-a)(a+b)(b+c)=(a-b)(b-c)(a-c)$

Thấy: $(a-b)(b+c)(c+a)+(b-c)(a+b)(c+a)+(c-a)(a+b)(b+c)=(a-b)(b+c)(c+a)+(b-c)(a+b)(c+a)+(c-b+b-a)(a+b)(b+c)=(a-b)(b+c)(c-b)+(b-c)(a+b)(a-b)=(a-b)(b-c)(a-c)$

Vậy (1) được CM.

Bây giờ ta cần CM:

$|\frac{(a-b)(b-c)(a-c)}{(a+b)(b+c)(c+a)}|<\frac{1}{8}\Leftrightarrow \frac{|a-b|.|b-c|.|a-c|}{(a+b)(b+c)(c+a)}<\frac{1}{8}$

Do $a;b;c$ là 3 cạnh một tam giác nên:

 

$\left\{\begin{matrix}|a-b|<c  &  & \\ |b-c|<a  &  & \\ |a-c|<b \end{matrix}\right.\Rightarrow |a-b|.|b-c|.|a-c|<abc$
Mà $(a+b)(b+c)(c+a)\geq 8abc$ (Cauchy 3 lần 2 số)$\Rightarrow \frac{|a-b|.|b-c|.|a-c|}{(a+b)(b+c)(c+a)}<\frac{abc}{8abc}=\frac{1}{8}$
Dấu = xảy ra khi: $a=b=c$

 

mình ko hiểu những gì bạn làm lắm

Bạn có đang học lớp 9 không, bạn không hiểu chỗ nào?

Ý b là $(a-b)^2(a^2+ab+b^2)\geq 0$ nhé.


Bài viết đã được chỉnh sửa nội dung bởi Viet Hoang 99: 16-02-2014 - 16:20


#187
Viet Hoang 99

Viet Hoang 99

    $\textbf{Trương Việt Hoàng}$

  • Điều hành viên THPT
  • 2291 Bài viết

66) Cho $a;b;c$ là 3 cạnh một tam giác có chu vi bằng $6$. Tìm Min $A=3(a^2+b^2+c^2)+2abc$
 

67) Cho $a;b;c$ là 3 cạnh một tam giác có chu vi bằng $2$. Tìm Min $B=4(a^3+b^3+c^3)+15abc$

 

68) Cho $a;b;c$ là 3 cạnh một tam giác có chu vi bằng $1$. Tìm Min $C=a^3+b^3+c^3+3abc$

 

69) Cho $a;b;c>0$ thỏa: $12\sum \frac{1}{a^2}=3+\sum \frac{1}{a}$. Cmr: $\sum \frac{1}{4a+b+c}\leq \frac{1}{6}$


Bài viết đã được chỉnh sửa nội dung bởi Viet Hoang 99: 17-02-2014 - 19:17


#188
angleofdarkness

angleofdarkness

    Thượng sĩ

  • Thành viên
  • 246 Bài viết

66) Cho $a;b;c$ là 3 cạnh một tam giác có chu vi bằng $6$. Tìm Min $A=3(a^2+b^2+c^2)+2abc$

 

Sao không phải là max????



#189
lahantaithe99

lahantaithe99

    Trung úy

  • Thành viên
  • 883 Bài viết

66) Cho $a;b;c$ là 3 cạnh một tam giác có chu vi bằng $6$. Tìm Min $A=3(a^2+b^2+c^2)+2abc$
 

 

Ta có $abc\geq (a+b-c)(a+c-b)(b+c-a)=(6-2a)(6-2b)(6-2b)$

$=216-72(a+b+c)+24(ab+bc+ac)-8abc=24(ab+bc+ac)-216-8abc$

$\Rightarrow 9abc\geq 24(ab+bc+ac)-216\Leftrightarrow 2abc\geq \frac{16}{3}(ab+bc+ac)-48$

$\Rightarrow A\geq 3(a^2+b^2+c^2)+\frac{16}{3}(ab+bc+ac)-48$

$=\frac{a^2+b^2+c^2}{3}+\frac{8}{3}(a+b+c)^2-48$

$\geq 4+96-48=52$



#190
angleofdarkness

angleofdarkness

    Thượng sĩ

  • Thành viên
  • 246 Bài viết

Ta có $abc\geq (a+b-c)(a+c-b)(b+c-a)=(6-2a)(6-2b)(6-2b)$

$=216-72(a+b+c)+24(ab+bc+ac)-8abc=24(ab+bc+ac)-216-8abc$

$\Rightarrow 9abc\geq 24(ab+bc+ac)-216\Leftrightarrow 2abc\geq \frac{16}{3}(ab+bc+ac)-48$

$\Rightarrow A\geq 3(a^2+b^2+c^2)+\frac{16}{3}(ab+bc+ac)-48$

$=\frac{a^2+b^2+c^2}{3}+\frac{8}{3}(a+b+c)^2-48$

$\geq 4+96-48=52$

 

Cách 2 bài 66/ 

 

Có thể dùng Cauchy: 8ebbb71b05e6930768bcb95223694c6b030c399b



#191
angleofdarkness

angleofdarkness

    Thượng sĩ

  • Thành viên
  • 246 Bài viết

67) Cho $a;b;c$ là 3 cạnh một tam giác có chu vi bằng $2$. Tìm Min $B=4(a^3+b^3+c^3)+15abc$

 

 

Bài này có nhiều cách, cách dùng BĐT Schur bậc 3:

 

$a^3+b^3+c^3+3abc \geq ab(a+b)+bc(b+c)+ca(c+a)$
 

$\Rightarrow 3(a^3+b^3+c^3)+9abc \geq 3[ab(a+b)+bc(b+c)+ca(c+a)]$

 

$\Rightarrow 4(a^3+b^3+c^3)+15abc \geq 3[ab(a+b)+bc(b+c)+ca(c+a)]+(a^3+b^3+c^3)+6abc=(a+b+c)^3$

 

(Vì có $(a+b+c)^3=a^3+b^3+c^3+3(a+b)(b+c)(c+a)$)

 

Hay $B \geq 8.$

 

Dấu = khi a = b = c = 2.


Bài viết đã được chỉnh sửa nội dung bởi angleofdarkness: 16-02-2014 - 18:41


#192
angleofdarkness

angleofdarkness

    Thượng sĩ

  • Thành viên
  • 246 Bài viết

68) Cho $a;b;c$ là 3 cạnh một tam giác có chu vi bằng $1$. Tìm Min $C=a^3+b^3+c^3+3abc$

 

 

Áp dụng BĐT $(a+b-c)(b+c-a)(c+a-b) \leq abc$
 
$\Leftrightarrow (1-2a)(1-2b)(1-2c) \leq abc \Leftrightarrow abc \geq -1+a(ab+bc+ca)-8abc$ (phá ngoặc)
 
$\Leftrightarrow 6abc \geq \frac{-2}{3}+\frac{8}{3}(ab+bc+ca)$ (1)
 
Mà $a^3+b^3+c^3-3abc=(a+b+c)(a^2+b^2+c^2-ab-bc-ca)$ nên $C=a^3+b^3+c^3+3abc \geq a^2+b^2+c^2+\frac{5}{3}(ab+bc+ca)-\frac{2}{3} \\ =\frac{1}{6}(a^2+b^2+c^2)+\frac{1}{6} \geq \frac{2}{9}$

Bài viết đã được chỉnh sửa nội dung bởi angleofdarkness: 16-02-2014 - 18:54


#193
lahantaithe99

lahantaithe99

    Trung úy

  • Thành viên
  • 883 Bài viết

 

68) Cho $a;b;c$ là 3 cạnh một tam giác có chu vi bằng $1$. Tìm Min $C=a^3+b^3+c^3+3abc$

 

 

Áp dụng tương tự bài 66 $3abc\geq \frac{4}{3}(ab+bc+ac)-\frac{1}{3}$

$\Rightarrow C\geq a^3+b^3+c^3+\frac{4}{3}(ab+bc+ac)-\frac{1}{3}$

Bằng $AM-GM$ có $a^3+b^3+c^3\geq\sum\frac{2a^2}{3}-\frac{1}{9}$

$\Rightarrow C\geq \sum\frac{2a^2}{3}+\frac{4}{3}(ab+bc+ac)-\frac{4}{9}=\frac{2}{3}-\frac{4}{9}=\frac{2}{9}$



#194
hoctrocuanewton

hoctrocuanewton

    Thiếu úy

  • Thành viên
  • 710 Bài viết

 

69) Cho $a;b;c>0$ thỏa: $12\sum \frac{1}{a^2}=3+\sum \frac{1}{a}$. Cmr: $\sum \frac{1}{4a+b+c}\leq \frac{1}{6}$

áp dụng bđt cô si ta có :

$12\sum (\frac{1}{a^{2}}+\frac{1}{9})\geqslant 12.\sum \frac{2}{3a}= 8\sum \frac{1}{a}$

theo đk bài toán ta có

$12\sum (\frac{1}{a^{2}}+\frac{1}{9})=7+\sum \frac{1}{a}\geqslant 8\sum \frac{1}{a}\Rightarrow \sum \frac{1}{a}\leqslant 1$

áp dụng bđt schwars ta có

$\sum (\frac{1}{4a+b+c})\leq \frac{1}{36}\sum (\frac{4}{a}+\frac{1}{b}+\frac{1}{c})=\frac{1}{36}\sum \frac{6}{a}\leq \frac{1}{6}$

vậy được đpcm



#195
lahantaithe99

lahantaithe99

    Trung úy

  • Thành viên
  • 883 Bài viết

 

 

69) Cho $a;b;c>0$ thỏa: $12\sum \frac{1}{a^2}=3+\sum \frac{1}{a}$. Cmr: $\sum \frac{1}{4a+b+c}\leq \frac{1}{6}$

Áp dụng $AM-GM$

$\frac{1}{a^2}+\frac{1}{9}\geq \frac{2}{3a}$

$\Rightarrow \sum \frac{1}{a^2}\geq \sum \frac{2}{3a}-\frac{1}{3}$

$\Rightarrow \sum \frac{12}{a^2}\geq \sum \frac{8}{a}-4$

$\Rightarrow 3+\sum \frac{1}{a}\geq \sum \frac{8}{a}-4\Leftrightarrow \sum\frac{1}{a}\leq 1$

Áp dụng bđt S.Vacxơ

$\frac{1}{4a+b+c}\leq \frac{1}{36}(\frac{4}{a}+\frac{1}{b}+\frac{1}{c})$

CMTT $\sum \frac{1}{4a+b+c}\leq \frac{1}{36}(\frac{6}{a}+\frac{6}{b}+\frac{6}{c})\leq \frac{1}{6}$



#196
Viet Hoang 99

Viet Hoang 99

    $\textbf{Trương Việt Hoàng}$

  • Điều hành viên THPT
  • 2291 Bài viết

Cách 2 bài 66/ 

 

Có thể dùng Cauchy: 8ebbb71b05e6930768bcb95223694c6b030c399b

Khó hiểu???
 

 

Áp dụng $AM-GM$

$\frac{1}{a^2}+\frac{1}{9}\geq \frac{2}{3a}$

$\Rightarrow \sum \frac{1}{a^2}\geq \sum \frac{2}{3a}-\frac{1}{3}$

$\Rightarrow \sum \frac{12}{a^2}\geq \sum \frac{8}{a}-4$

$\Rightarrow 3+\sum \frac{1}{a}\geq \sum \frac{8}{a}-4\Leftrightarrow \sum\frac{1}{a}\leq 1$

Áp dụng bđt S.Vacxơ

$\frac{1}{4a+b+c}\leq \frac{1}{36}(\frac{4}{a}+\frac{1}{b}+\frac{1}{c})$

CMTT $\sum \frac{1}{4a+b+c}\leq \frac{1}{36}(\frac{6}{a}+\frac{6}{b}+\frac{6}{c})\leq \frac{1}{6}$

 

 

áp dụng bđt cô si ta có :

$12\sum (\frac{1}{a^{2}}+\frac{1}{9})\geqslant 12.\sum \frac{2}{3a}= 8\sum \frac{1}{a}$

theo đk bài toán ta có

$12\sum (\frac{1}{a^{2}}+\frac{1}{9})=7+\sum \frac{1}{a}\geqslant 8\sum \frac{1}{a}\Rightarrow \sum \frac{1}{a}\leqslant 1$

áp dụng bđt schwars ta có

$\sum (\frac{1}{4a+b+c})\leq \frac{1}{36}\sum (\frac{4}{a}+\frac{1}{b}+\frac{1}{c})=\frac{1}{36}\sum \frac{6}{a}\leq \frac{1}{6}$

vậy được đpcm

69)
Cách 2:

$3\sum \frac{1}{a^2}\geq (\sum \frac{1}{a})^2\Leftrightarrow 12\sum \frac{1}{a^2}\geq 4(\sum \frac{1}{a})^2\Rightarrow 3+\sum \frac{1}{a}\geq 4(\sum \frac{1}{a})^2$

Đặt $\sum \frac{1}{a}=t$

$\Rightarrow 3+t\geq 4t^2\Leftrightarrow t\leq 1$

hay $\sum \frac{1}{a}\leq 1$

Có: $\frac{4}{a}+\frac{1}{b}+\frac{1}{c}\geq \frac{36}{4a+b+c}$ (Áp dụng BCS dạng cộng mẫu cho 6 số)
$\Rightarrow \sum \frac{1}{a}\geq 6\sum \frac{1}{4a+b+c}\Rightarrow \frac{1}{4a+b+c}\leq \frac{1}{6}.\sum \frac{1}{a}\leq \frac{1}{6}$



#197
angleofdarkness

angleofdarkness

    Thượng sĩ

  • Thành viên
  • 246 Bài viết

Khó hiểu???

 

Biến đổi tương đương, hơi dài nên không buồn đánh latex ra :D



#198
Viet Hoang 99

Viet Hoang 99

    $\textbf{Trương Việt Hoàng}$

  • Điều hành viên THPT
  • 2291 Bài viết

70) Cho $x;y>0$ thỏa: $(x+y-1)^2=xy$. Tìm Min $P=\frac{1}{xy}+\frac{1}{x^2+y^2}+\frac{\sqrt{xy}}{x+y}$

 

71) Cho $a;b;c$ là 3 cạnh một tam giác. Cmr: $1< \sum \frac{a}{b+c}<2$


Bài viết đã được chỉnh sửa nội dung bởi Viet Hoang 99: 22-02-2014 - 19:43


#199
angleofdarkness

angleofdarkness

    Thượng sĩ

  • Thành viên
  • 246 Bài viết

71) Cho $a;b;c$ là 3 cạnh một tam giác. Cmr: $1\leq \sum \frac{a}{b+c}<2$

 

 

Do a, b, c là độ dài ba cạnh tam giác nên ta có $\frac{a}{a+b+c}<\frac{a}{b+c}<\frac{a+c}{a+b+c}$

 

$\Rightarrow \sum \frac{a}{a+b+c}<\sum \frac{a}{b+c}<\sum \frac{a+c}{a+b+c}$

 

Hay $1<\sum \frac{a}{b+c}<2$



#200
Viet Hoang 99

Viet Hoang 99

    $\textbf{Trương Việt Hoàng}$

  • Điều hành viên THPT
  • 2291 Bài viết

 

8)

$S=\sum a\sqrt[3]{b^{2}+c^{2}}=\sum \sqrt[3]{a^{3}(b^{2}+c^{2})}=\sum \sqrt[6]{(a^{2})^{3}.(b^{2}+c^{2})^{2}}=\sum \sqrt[6]{(2a^{2})^{3}.(b^{2}+c^{2})^{2}.8}.\sqrt[6]{\frac{1}{64}}\leq \sum \frac{2a^{2}.3+(b^{2}+c^{2}.2+8)}{6}.\sqrt[6]{\frac{1}{64}}=\sum \frac{6a^{2}+2b^{2}+2c^{2}+8}{6}.\sqrt[6]{\frac{1}{64}}=\frac{10(a^{2}+b^{2}+c^{2})+24}{6}=\sqrt[6]{\frac{1}{64}}=12$

Dấu = xảy ra khi: $a=b=c=2$

 

 

Mình đã làm rồi mà.






1 người đang xem chủ đề

0 thành viên, 1 khách, 0 thành viên ẩn danh